- Fri Nov 03, 2017 3:13 pm
#41178
Complete Question Explanation
(The complete setup for this game can be found here: lsat/viewtopic.php?t=15645)
The correct answer choice is (C)
Answer choice (A) is incorrect because only H, L, P, G, or T can be harvested first.
Answer choice (B) is incorrect because the earliest K can be harvested is fourth.
Answer choice (D) is incorrect because the latest G can be harvested is sixth.
Answer choice (E) is incorrect because only M or J can be harvested last.
(The complete setup for this game can be found here: lsat/viewtopic.php?t=15645)
The correct answer choice is (C)
Answer choice (A) is incorrect because only H, L, P, G, or T can be harvested first.
Answer choice (B) is incorrect because the earliest K can be harvested is fourth.
Answer choice (D) is incorrect because the latest G can be harvested is sixth.
Answer choice (E) is incorrect because only M or J can be harvested last.